公立諏訪東京理科大学2023年前期第2問

公立諏訪【前期】
スポンサーリンク

公立諏訪東京理科大学2023年前期第2問の問題文全文

関数 \(f(\theta )=\sin^3{\theta}+\cos^3{\theta}\) について\(,\) 以下の問いに答えよ.

(1) \(u=\sin{\theta}+\cos{\theta}\) とおいて\(,\) \(f(\theta )\) を \(u\) の式で表せ.

(2) \(u\) のとり得る値の範囲を求めよ.

(3) \(f(\theta )\) の最大値と最小値を求めよ.

(4) 次の積分の値を求めよ.

\begin{align}\int_0^{\pi}|f(\theta )|d\theta \end{align}

(1) の解答〜対称式を基本対称式で表す〜

\(u=\sin{\theta}+\cos{\theta}\) より\(,\)

\begin{align}u^2=\sin^2{\theta}+2\sin{\theta}\cos{\theta}+\cos^2{\theta}=1+2\sin{\theta}\cos{\theta}\end{align}

\begin{align}\sin{\theta}\cos{\theta}=\frac{1}{2}(u^2-1)\end{align}

quandle
quandle

\(f(\theta )\) は \(\sin{\theta}\) と \(\cos{\theta}\) に関する対称式です. すべての対称式は基本対称式だけで表せますから\(,\) まずは基本対称式の \(\sin{\theta}\cos{\theta}\) を準備しましょう.

\begin{align}f(\theta )=(\sin{\theta}+\cos{\theta})^3-3\sin{\theta}\cos{\theta}(\sin{\theta}+\cos{\theta})\end{align}

\begin{align}=u^3-\frac{3}{2}(u^2-1)u=-\frac{1}{2}u^3+\frac{3}{2}u~~~~\cdots \fbox{答}\end{align}

(2) の解答〜 \(\sin{\theta}\) と \(\cos{\theta}\) の1次結合は合成〜

\begin{align}u=\sqrt{2}\sin{\left(\theta +\frac{\pi}{4}\right)}\end{align}

\(\displaystyle \theta+\frac{\pi}{4}\) はすべての実数値をとるので\(,\)

\begin{align}-1\leqq \sin{\left(\theta +\frac{\pi}{4}\right)}\leqq 1\end{align}

\begin{align}\therefore -\sqrt{2}\leqq u \leqq \sqrt{2}~~~~\cdots \fbox{答}\end{align}

(3) の解答〜 \(3\) 次関数の最大最小は微分で〜

\begin{align}g(u)=-\frac{1}{2}u^3+\frac{3}{2}u\end{align}

とおく.

\begin{align}g^{\prime}(u)=-\frac{3}{2}u^2+\frac{3}{2}=-\frac{3}{2}(u-1)(u+1)\end{align}

\(-\sqrt{2}\leqq u \leqq \sqrt{2}\) における増減表は以下のようになる.

\begin{align}\begin{array}{|c|c|c|c|c|c|c|c|}\hline u & -\sqrt{2} & \cdots & -1 & \cdots & 1 & \cdots & \sqrt{2}\\ \hline g^{\prime}(u) & {} & – & 0 & + & 0 & – & {} \\ \hline g(u) & \displaystyle -\frac{1}{\sqrt{2}} & \searrow & -1 & \nearrow & 1 & \searrow & \displaystyle \frac{1}{\sqrt{2}}\\ \hline \end{array}\end{align}

増減表より\(,\)

\begin{align}最大値~1,~最小値~-1~~~~\cdots \fbox{ 答}\end{align}

quandle
quandle

単に最大値と最小値を求めよとしか言っていないのでこれでOKです!

補足1〜最大値・最小値をとる \(\theta \) の値を求める〜

問題では聞かれていませんがもし

最大値・最小値およびそのときの \(\theta \) の値を求めよ

と聞かれた場合の解答も載せておきます. 問題文には \(\theta \) の範囲について言及がないので\(,\) こういう場合はすべての実数をとるものとして\(,\) 一般角で答えるようにしましょう.

【解答】

(先程の)増減表より\(,\) \(u=1\) のとき最大値 \(1\) をとる. このときの \(\theta \) の値は

\begin{align}u=1\Leftrightarrow \sin{\left(\theta +\frac{\pi}{4}\right)}=\frac{1}{\sqrt{2}}\end{align}

整数 \(n\) を用いて\(,\)

\begin{align}\theta +\frac{\pi}{4}=\frac{\pi}{4}+2n\pi~\mathrm{or}~\frac{3}{4}\pi+2n\pi\end{align}

\begin{align}\theta =2n\pi~\mathrm{or}~\left(2n+\frac{1}{2}\right)\pi~~~~\cdots \fbox{答}\end{align}

また\(,\) \(u=-1\) のとき最小値 \(-1\) をとる. このときの \(\theta \) の値は

\begin{align}u=-1\Leftrightarrow \sin{\left(\theta +\frac{\pi}{4}\right)}=-\frac{1}{\sqrt{2}}\end{align}

\begin{align}\theta +\frac{\pi}{4}=\frac{5}{4}\pi +2n\pi~\mathrm{or}~\frac{7}{4}\pi+2n\pi\end{align}

\begin{align}\therefore \theta =(2n+1)\pi~\mathrm{or}~\left(2n+\frac{3}{2}\right)\pi\end{align}

補足2〜奇関数なので増減表を半分にできる〜

\begin{align}g(-u)=-\frac{1}{2}(-u)^3+\frac{3}{2}(-u)=\frac{1}{2}u^3-\frac{3}{2}u\end{align}

\begin{align}=-\left(-\frac{1}{2}u^3+\frac{3}{2}u\right)=-g(u)\end{align}

より\(,\) \(g(u)\) は奇関数である. よって\(,\) \(-\sqrt{2}\leqq u \leqq 0\) におけるグラフは\(,\) \(0\leqq u \leqq \sqrt{2}\) におけるグラフを原点対称にしたものになる.

quandle
quandle

原点対称のグラフということがわかるので\(,\) \(0\leqq u \leqq \sqrt{2}\) の部分だけ増減表をかけばいいので労力を半分に減らすことができます. ただ\(,\) 本問の場合は全体をかいても大した労力にはならないのと\(,\) 原点対称により符号ミスが起こる可能性が高まるリスクを秘めていることなど考えて\(,\) あえて全体の増減表をかいています.

(4) の解答〜 絶対値を適切に外す〜

\(f(\theta )\) の符号を調べる

quandle
quandle

絶対値つきの関数は中身が正ならそのまま外れ\(,\) 中身が負ならマイナスをつけて外すことになります. まずは \(f(\theta )\) の符号を調べることから始めましょう!

\(\displaystyle 0\leqq \theta \leqq \frac{\pi}{2}\) のとき\(,\) \(\sin{\theta}>0\) かつ \(\cos{\theta}>0\) なので\(,\) 明らかに \(f(\theta )>0\) である. \(\displaystyle \frac{\pi}{2}\leqq \theta \leqq \pi \) における符号を調べればよい.

\begin{align}f^{\prime}(\theta )=3\sin^2{\theta}\cos{\theta}-3\cos^2{\theta}\sin{\theta}\end{align}

\begin{align}=3\sin{\theta}\cos{\theta}(\sin{\theta}-\cos{\theta})\end{align}

\(\displaystyle \frac{\pi}{2}\leqq \theta \leqq \pi\) のとき\(,\) \(\sin{\theta}>0,~\cos{\theta}<0\) より\(,\)

\begin{align}3\sin{\theta}\cos{\theta}<0~ かつ~ \sin{\theta}-\cos{\theta}>0\end{align}

であるから\(,\)

\begin{align}f^{\prime}(\theta )<0\end{align}

なので\(,\) \(f(\theta )\) は \(\displaystyle \frac{\pi}{2}\leqq \theta \leqq \pi\) において単調減少である.

\(f(\theta )\) は連続関数であり\(,\)

\begin{align}f\left(\frac{\pi}{2}\right)=1>0,~f(\pi )=-1<0\end{align}

であるから\(,\) 中間値の定理により \(f(\theta )=0\) となる \(\theta \) は \(\displaystyle \frac{\pi}{2}<\theta <\pi\) の範囲にただ一つ存在する. そしてその \(\theta \) の値は \(\displaystyle \theta =\frac{3}{4}\pi \) である.

quandle
quandle

\(f(\theta )=0\) となる \(\theta \) は \(\sin{\theta}\) と \(\cos{\theta}\) が逆符号になるものを見つけてくればいいのですが\(,\) それはみんな暗記していると思います.

\begin{align}\sin{\frac{3}{4}\pi}=\frac{1}{\sqrt{2}},~\cos{\frac{3}{4}\pi}=-\frac{1}{\sqrt{2}}\end{align}

はおなじみですね. 本来は \(f(\theta )=0\) の方程式を解いて \(\theta \) を求めに行くのでしょうが\(,\) 今回の場合は \(f(\theta )=0\) となる \(\theta \) はただ \(1\) つしかないことを示していて\(,\) \(f(\theta )=0\) となる \(\theta \) を具体的に探してきたということなので解答として十分なものになっています.

以上より\(,\)

\begin{align} 0\leqq \theta \leqq \frac{3}{4}\pi ~のとき~f(\theta )>0\end{align}

\begin{align}\frac{3}{4}\pi \leqq \theta \pi~のとき~f(\theta )<0\end{align}

\(\displaystyle F(\theta )=\int f(\theta )d\theta \) を求める

\begin{align}F(\theta )=\int (\sin^3{\theta}+\cos^3{\theta})d\theta \end{align}

とおく. \(C,~C_1,~C_2\) を任意の実数とする.

\begin{align}\int \sin^3{\theta}d\theta =\int \sin{\theta}(1-\cos^2{\theta})d\theta \end{align}

\begin{align}=-\cos{\theta}+\frac{1}{3}\cos^3{\theta}+C_1\end{align}

\begin{align}\int \cos^3{\theta}d\theta =\int \cos{\theta}(1-\sin^2{\theta})d\theta \end{align}

\begin{align}=\sin{\theta}-\frac{1}{3}\sin^3{\theta}+C_2\end{align}

よって\(,\)

\begin{align}F(\theta )=\cos{\theta}+\frac{1}{3}\cos^3{\theta}+\sin{\theta}-\frac{1}{3}\sin^3{\theta}+C\end{align}

以上で与式(\(=I\)とおく)を計算する準備は整った.

与式(\(=I\)とおく)の値を求める

\begin{align}I=\int_0^{\pi}|f(\theta )|d\theta=\int_0^{\frac{3}{4}\pi}f(\theta )d\theta -\int_{\frac{3}{4}\pi}^{\pi}f(\theta )d\theta \end{align}

\begin{align}=\biggl[F(\theta )\biggr]_0^{\frac{3}{4}\pi}-\biggl[F(\theta )\biggr]_{\frac{3}{4}\pi}^{\pi}=2F\left(\frac{3}{4}\pi \right)-F(0)-F(\pi )\end{align}

ここで\(,\)

\begin{align}F\left(\frac{3}{4}\pi \right)=\frac{1}{\sqrt{2}}+\frac{1}{3}\left(-\frac{1}{2\sqrt{2}}\right)+\frac{1}{\sqrt{2}}-\frac{1}{3}\left(\frac{1}{2\sqrt{2}}\right)=\frac{5\sqrt{2}}{6}\end{align}

\begin{align}F(0)=-1+\frac{1}{3}=-\frac{2}{3},~F(\pi )=1-\frac{1}{3}=\frac{2}{3}\end{align}

であるから\(,\)

\begin{align}I=2\cdot \frac{5\sqrt{2}}{6}-\left(-\frac{2}{3}\right)-\frac{2}{3}=\frac{5\sqrt{2}}{3}~~~~\cdots \fbox{答}\end{align}

コメント

タイトルとURLをコピーしました